Question and Answers Forum

All Questions      Topic List

Arithmetic Questions

Previous in All Question      Next in All Question      

Previous in Arithmetic      Next in Arithmetic      

Question Number 167776 by peter frank last updated on 24/Mar/22

Answered by MikeH last updated on 24/Mar/22

ax^2 +bx +c = a(x^2 +(b/a)x + (c/a))                          = a[(x+(b/(2a)))^2 −(b^2 /(4a^2 ))+(c/a)]                          = a[(x+(b/(2a)))^2 +((−b^2 +4ac)/(4a^2 ))]                          = a(x+(b/(2a)))^2 +((4ac−b^2 )/(4a))  ∫(1/(a(x+(b/(2a)))^2 +((4ac−b^2 )/(4a)))) dx  Let u = x + (b/(2a)) ⇒ du = dx  ∫(1/(au^2 +k))du ,  where k = ((4ac−b^2 )/(4a))  ∫(1/(a(u^2 +((√(k/a)))^2 )))du = (1/a)(√(a/k)) tan^(−1) (x(√(a/k))) +p.

$${ax}^{\mathrm{2}} +{bx}\:+{c}\:=\:{a}\left({x}^{\mathrm{2}} +\frac{{b}}{{a}}{x}\:+\:\frac{{c}}{{a}}\right) \\ $$$$\:\:\:\:\:\:\:\:\:\:\:\:\:\:\:\:\:\:\:\:\:\:\:\:=\:{a}\left[\left({x}+\frac{{b}}{\mathrm{2}{a}}\right)^{\mathrm{2}} −\frac{{b}^{\mathrm{2}} }{\mathrm{4}{a}^{\mathrm{2}} }+\frac{{c}}{{a}}\right] \\ $$$$\:\:\:\:\:\:\:\:\:\:\:\:\:\:\:\:\:\:\:\:\:\:\:\:=\:{a}\left[\left({x}+\frac{{b}}{\mathrm{2}{a}}\right)^{\mathrm{2}} +\frac{−{b}^{\mathrm{2}} +\mathrm{4}{ac}}{\mathrm{4}{a}^{\mathrm{2}} }\right] \\ $$$$\:\:\:\:\:\:\:\:\:\:\:\:\:\:\:\:\:\:\:\:\:\:\:\:=\:{a}\left({x}+\frac{{b}}{\mathrm{2}{a}}\right)^{\mathrm{2}} +\frac{\mathrm{4}{ac}−{b}^{\mathrm{2}} }{\mathrm{4}{a}} \\ $$$$\int\frac{\mathrm{1}}{{a}\left({x}+\frac{{b}}{\mathrm{2}{a}}\right)^{\mathrm{2}} +\frac{\mathrm{4}{ac}−{b}^{\mathrm{2}} }{\mathrm{4}{a}}}\:{dx} \\ $$$$\mathrm{Let}\:{u}\:=\:{x}\:+\:\frac{{b}}{\mathrm{2}{a}}\:\Rightarrow\:{du}\:=\:{dx} \\ $$$$\int\frac{\mathrm{1}}{{au}^{\mathrm{2}} +{k}}{du}\:,\:\:\mathrm{where}\:{k}\:=\:\frac{\mathrm{4}{ac}−{b}^{\mathrm{2}} }{\mathrm{4}{a}} \\ $$$$\int\frac{\mathrm{1}}{{a}\left({u}^{\mathrm{2}} +\left(\sqrt{\frac{{k}}{{a}}}\right)^{\mathrm{2}} \right)}{du}\:=\:\frac{\mathrm{1}}{{a}}\sqrt{\frac{{a}}{{k}}}\:\mathrm{tan}^{−\mathrm{1}} \left({x}\sqrt{\frac{{a}}{{k}}}\right)\:+{p}. \\ $$$$ \\ $$

Commented by peter frank last updated on 26/Mar/22

thank you

$$\mathrm{thank}\:\mathrm{you} \\ $$

Answered by MJS_new last updated on 25/Mar/22

∫(dx/(ax^2 +bx+c))=       [t=2ax+b → dx=(dt/(2a))]  =2∫(dt/(t^2 +4ac−b^2 ))  (1) 4ac−b^2 <0 ⇒ 4ac−b^2 =−d^2 ∧d>0  2∫(dt/(t^2 +4ac−b^2 ))=2∫(dt/(t^2 −d^2 ))=2∫(dt/((t−d)(t+d)))=  =(1/d)∫((1/(t−d))−(1/(t+d)))dt=((ln (t−d) −ln (t+d))/d)=  =((ln ∣2ax+b−(√(b^2 −4ac))∣ −ln ∣2ax+b+(√(b^2 −4ax))∣)/( (√(b^2 −4ac))))+C  (2) 4ac−b^2 =0  2∫(dt/t^2 )=−(2/t)=−(2/(2ax+b))+C  (3) 4ac−b^2 >0 ⇒ 4ac−b^2 =d^2 ∧d>0  2∫(dt/(t^2 +4ac−b^2 ))=2∫(dt/(t^2 +d^2 ))=  =((2arctan (t/d))/d)=  =((2arctan ((2ax+b)/( (√(4ac−b^2 )))))/( (√(4ac−b^2 ))))+C

$$\int\frac{{dx}}{{ax}^{\mathrm{2}} +{bx}+{c}}= \\ $$$$\:\:\:\:\:\left[{t}=\mathrm{2}{ax}+{b}\:\rightarrow\:{dx}=\frac{{dt}}{\mathrm{2}{a}}\right] \\ $$$$=\mathrm{2}\int\frac{{dt}}{{t}^{\mathrm{2}} +\mathrm{4}{ac}−{b}^{\mathrm{2}} } \\ $$$$\left(\mathrm{1}\right)\:\mathrm{4}{ac}−{b}^{\mathrm{2}} <\mathrm{0}\:\Rightarrow\:\mathrm{4}{ac}−{b}^{\mathrm{2}} =−{d}^{\mathrm{2}} \wedge{d}>\mathrm{0} \\ $$$$\mathrm{2}\int\frac{{dt}}{{t}^{\mathrm{2}} +\mathrm{4}{ac}−{b}^{\mathrm{2}} }=\mathrm{2}\int\frac{{dt}}{{t}^{\mathrm{2}} −{d}^{\mathrm{2}} }=\mathrm{2}\int\frac{{dt}}{\left({t}−{d}\right)\left({t}+{d}\right)}= \\ $$$$=\frac{\mathrm{1}}{{d}}\int\left(\frac{\mathrm{1}}{{t}−{d}}−\frac{\mathrm{1}}{{t}+{d}}\right){dt}=\frac{\mathrm{ln}\:\left({t}−{d}\right)\:−\mathrm{ln}\:\left({t}+{d}\right)}{{d}}= \\ $$$$=\frac{\mathrm{ln}\:\mid\mathrm{2}{ax}+{b}−\sqrt{{b}^{\mathrm{2}} −\mathrm{4}{ac}}\mid\:−\mathrm{ln}\:\mid\mathrm{2}{ax}+{b}+\sqrt{{b}^{\mathrm{2}} −\mathrm{4}{ax}}\mid}{\:\sqrt{{b}^{\mathrm{2}} −\mathrm{4}{ac}}}+{C} \\ $$$$\left(\mathrm{2}\right)\:\mathrm{4}{ac}−{b}^{\mathrm{2}} =\mathrm{0} \\ $$$$\mathrm{2}\int\frac{{dt}}{{t}^{\mathrm{2}} }=−\frac{\mathrm{2}}{{t}}=−\frac{\mathrm{2}}{\mathrm{2}{ax}+{b}}+{C} \\ $$$$\left(\mathrm{3}\right)\:\mathrm{4}{ac}−{b}^{\mathrm{2}} >\mathrm{0}\:\Rightarrow\:\mathrm{4}{ac}−{b}^{\mathrm{2}} ={d}^{\mathrm{2}} \wedge{d}>\mathrm{0} \\ $$$$\mathrm{2}\int\frac{{dt}}{{t}^{\mathrm{2}} +\mathrm{4}{ac}−{b}^{\mathrm{2}} }=\mathrm{2}\int\frac{{dt}}{{t}^{\mathrm{2}} +{d}^{\mathrm{2}} }= \\ $$$$=\frac{\mathrm{2arctan}\:\frac{{t}}{{d}}}{{d}}= \\ $$$$=\frac{\mathrm{2arctan}\:\frac{\mathrm{2}{ax}+{b}}{\:\sqrt{\mathrm{4}{ac}−{b}^{\mathrm{2}} }}}{\:\sqrt{\mathrm{4}{ac}−{b}^{\mathrm{2}} }}+{C} \\ $$

Commented by peter frank last updated on 26/Mar/22

thank you

$$\mathrm{thank}\:\mathrm{you} \\ $$

Commented by MJS_new last updated on 27/Mar/22

hey Peter, you′re welcome! regards from  Vienna (Austria), Martin

$$\mathrm{hey}\:\mathrm{Peter},\:\mathrm{you}'\mathrm{re}\:\mathrm{welcome}!\:\mathrm{regards}\:\mathrm{from} \\ $$$$\mathrm{Vienna}\:\left(\mathrm{Austria}\right),\:\mathrm{Martin} \\ $$

Terms of Service

Privacy Policy

Contact: info@tinkutara.com